Answer:
$895.09
Step-by-step explanation:
Applying the given formula:
A = $600e^(0.04*10), or
= $895.09
We find that the answer is $895.09.
Compound interest is interest calculated on the initial principal, which also includes all of the accumulated interest from the previous.
How to find how much would $600 be worth after 10 years, if it were invested at 4% interest compounded continuously?
Hint: Use the formula below and round your answer to the nearest cent.)
A(t)=P•e^rt.
Applying the given formula:
A = $600e^(0.04*10),
or
= $895.09.
Learn more about references on: https://brainly.com/question/18456266
#SPJ2
What is the base if the rate is 41% on the percentage is 83
Answer:
202,44
Step-by-step explanation:
We know that 41% of a number is 83, and we need to find the base number. To do so, we're going to use the rule of three, as follows:
If 83 represents -----------------------------> 41% of a number
X <------------------------------ 100% of a number
Then:
X = (100 * 83)/41 = 202,44
Therefore, the base number is: 202,44 ✅
Which of the following represents the factorization of the trinomial below?
x^2 + 13x - 30
Answer:b
Step-by-step explanation:
The factors of the trinomial is option (D) (x+15)(x-2) is the correct answer.
What is factorization?Factorization is the breaking or decomposition of an entity (a number, a matrix, or a polynomial) into a product of another entity, or factors, which when multiplied together give the original number. Factorize an expression involves take out the greatest common factor (GCF) of all the terms.
For the given situation,
The trinomial is x^2 + 13x - 30.
The trinomial can be factored as
⇒ [tex]x^2 + 13x - 30=0[/tex]
⇒ [tex]x^2 + 15x-2x - 30=0[/tex]
⇒ [tex]x(x+15)-2(x+15)=0[/tex]
⇒ [tex](x+15)(x-2)=0[/tex]
Hence we can conclude that the factors of the trinomial is option (D) (x+15)(x-2) is the correct answer.
Learn more about factorization here
https://brainly.com/question/22048687
#SPJ2
Please help, math is my worst subject and I would really appreciate it!
Answer:
MW=CD
Step-by-step explanation:
In a soccer team, 1/3 of the players ride a bike to practice, 25% walk to practice and the remaining 15 players are driven by their parents. How many players are there in the team?
a. 42 b. 36 c. 30 d. 21
Final answer:
By setting up an equation based on the proportions of players who bike, walk, and are driven to practice, and solving for the total number of players, it is determined that there are 36 players on the soccer team.
Explanation:
To determine the total number of players in the soccer team, we can use the information given about the fractions of players commuting by different modes of transportation and the number of players who are driven by their parents.
We know that 1/3 of the players ride a bike, 25% (which is equivalent to 1/4) walk, and the remaining 15 players are driven by their parents.
Let's denote the total number of players as T. The parts of the team that ride a bike and walk can be represented as T/3 and T/4, respectively. The remaining players are represented by the number 15.
Since these components add up to the whole team, we can write the equation:
T/3 + T/4 + 15 = T
To solve for T, we first need to find a common denominator, which is 12 in this case. The equation becomes:
4T/12 + 3T/12 + 15 = T
Combining the T terms gives us:
7T/12 + 15 = T
This simplifies to:
7T/12 = T - 15
Multiplying both sides of the equation by 12 to eliminate the fraction gives us:
7T = 12T - 180
Subtracting 7T from both sides results in:
5T = 180
Dividing both sides by 5, we finally get:
T = 36
Therefore, there are 36 players in the soccer team.
Find x.
A.8
B.10
C.10.5
D.12
Answer:
10
Step-by-step explanation:
Setup a proportional for similar triangles when finding lengths.
We have FG~MN and GH~N0.
Corresponding sides of similar triangles are proportional.
[tex]\frac{FG}{MN}=\frac{GH}{NO}[/tex]
[tex]\frac{6}{x}=\frac{9}{15}[/tex]
Cross-multiply:
[tex]9x=6(15)[/tex]
Simplify right hand side:
[tex]9x=90[/tex]
Divide both sides by 9:
[tex]x=\frac{90}{9}[/tex]
Simplify right hand side:
[tex]x=10[/tex]
Cut 63 cm stick into two pieces in such a way that the first piece is 7 less than the second one. Let x be the second one.
Answer:
We have divided the bar into two parts. For example, x and y.
Step-by-step explanation:
So, we have
63cm=x+y (1)
But one of those parts is 7 centimeters smaller than the other.We take y as the smallest part.Then:
y=x-7cm (2)
we replace this information in the first equation
63cm=x+(x-7cm)
63cm=x+x-7cm
7 goes to add to the other side
63cm+7cm=2x
70cm=2x
2 goes to divide to the other side
35cm=x
from the second equation we have
y=x-7cm
y=35cm-7cm=28cm
Finally
x=35cm
y=28cm
x+y=35cm+28cm=63cm
Determine the equation of quadratic function represented by the table of value below.
Answer:
A
Step-by-step explanation:
The general rule for the quadratic function is
[tex]y=ax^2+bx+c[/tex]
Use the data from the table:
[tex]y(0)=6\Rightarrow 6=a\cdot 0^2+b\cdot 0+c\\ \\y(1)=4\Rightarrow 4=a\cdot 1^2+b\cdot 1+c\\ \\y(-1)=10\Rightarrow 10=a\cdot (-1)^2+b\cdot (-1)+c[/tex]
We get the system of three equations:
[tex]\left\{\begin{array}{l}c=6\\ \\a+b+c=4\\ \\a-b+c=10\end{array}\right.[/tex]
From the first equation
[tex]c=6,[/tex] then
[tex]\left\{\begin{array}{l}a+b=-2\\ \\a-b=4\end{array}\right.[/tex]
Add these two equations:
[tex]a+b+a-b=-2+4\\ \\2a=2\\ \\a=1[/tex]
So,
[tex]b=-2-a=-2-1=-3[/tex]
The quadratic function is
[tex]y=1\cdot x^2-3x+6\\ \\y=x^2-3x+6[/tex]
Nick mowed about 3/5 of the school lawn yesterday. The mowed another 1/4 of the remaining portion of the lawn this morning. How much is left to mow?
A. 1/4
B.2/5
C.3/5
D.3/4
E.19/20
Answer:
1/4
Step-by-step explanation:
3/5 x 1/4 = 3/20
3/20 + 2/5 = 3/4
1 - 3/4 = 1/4
so the answer is A
what is y=20^x in log
[tex]y=20^x\Longleftrightarrow x=\log_{20}y[/tex]
Examine the following system of inequalities.
{y > −x + 4 and y ≤−(1/2)^x + 6
Which graph shows the solution to the system?
Dotted linear inequality shaded below passes through (negative 4, 0) and (0, 4). Solid exponential inequality shaded above passes through (negative 1, 8) & (0, 7).
Dotted linear inequality shaded below passes through (0, 4) and (4, 0). Solid exponential inequality shaded below passes through (negative 2,2) & (0,5).
Dotted linear inequality shaded above passes through (negative 4, 0) and (0, 4). Solid exponential inequality shaded below passes through (negative 1, 8) & (0, 7).
Dotted linear inequality shaded above passes through (0, 4) and (4, 0). Solid exponential inequality shaded below passes through (negative 2,2) & (0,5).
Answer:
Dotted linear inequality shaded above passes through (0, 4) and (4, 0). Solid exponential inequality shaded below passes through (negative 2,2) & (0,5)
Step-by-step explanation:
we have
[tex]y > -x+4[/tex] ----> inequality A
The solution of the inequality A is the shaded area above the dotted line [tex]y=-x+4[/tex]
The dotted line passes through the points (0,4) and (4,0) (y and x-intercepts)
and
[tex]y \leq -(1/2)^{x} +6[/tex] -----> inequality B
The solution of the inequality B is the shaded area above the solid line [tex]y=-(1/2)^{x} +6[/tex]
The solid line passes through the points (0,5) and (-2,2)
therefore
The solution of the system of inequalities is the shaded area between the dotted line and the solid line
see the attached figure
Dotted linear inequality shaded above passes through (0, 4) and (4, 0). Solid exponential inequality shaded below passes through (negative 2,2) & (0,5)
Answer:
A i think
Step-by-step explanation:
Write
as a percentage.
For this case we must write as a percentage the following expression:
[tex]\frac {21} {25}[/tex]
Dividing we have to:
[tex]\frac {21} {25} = 0.84[/tex]
Now we multiply by 100%. So:
[tex]0.84 * 100 =[/tex]
We run the decimal two spaces to the right, finally we have:
84%
Answer:
84%
[tex]\text{Hey there!}[/tex]
[tex]\text{Percentages usually run out of 100}[/tex]
[tex]\dfrac{21}{25}\ = \ 21\div25\ = \ 0.84[/tex]
[tex]\huge\text{Decimal form: 0.84}[/tex]
[tex]\text{Remember what I said, percentages run out 100. ( e.g. x * 100)}[/tex]
[tex]\text{0.84\% }\times\text{ 100 = 84\%}[/tex]
[tex]\boxed{\boxed{\huge\text{Answer: 84\%}}}\huge\checkmark[/tex]
[tex]\text{Good luck on your assignment and enjoy your day!}[/tex]
~[tex]\frak{LoveYourselfFirst:)}[/tex]
Assume a normal distribution and that the average phone call in a certain town lasted 9 min, with a standard deviation of 1 min. What percentage of the calls lasted less than 8 min?
Answer:
The percentage of the calls lasted less than 8 min is 16%
Step-by-step explanation:
* Lets explain how to solve the problem
- To find the percentage of the calls lasted less than 8 min, find the
z-score for the calls lasted
∵ The rule of z-score is z = (x - μ)/σ , where
# x is the score
# μ is the mean
# σ is the standard deviation
* Lets solve the problem
- The average phone call in a certain town lasted is 9 min
∴ The mean (μ) = 9
- The standard deviation is 1 min
∴ σ = 1
- The calls lasted less than 8 min
∴ x = 8
∵ z = (x - μ)/σ
∴ z = (8 - 9)/1 = -1/1 = -1
∴ P(z < 8) = -1
- Use z-table to find the percentage of x < 8
∴ P(x < 8) = 0.15866 × 100% = 15.87% ≅ 16%
* The percentage of the calls lasted less than 8 min is 16%
Answer:
The percentage of the calls lasted less than 8 min is 16%.
Step-by-step explanation:
We are dealing with a normal distribution with an average phone call of 9 min and a standard deviation of 1 min. Below we can observe the empirical rule applied with a mean of 9 and a standard deviation of 1. The number 8 represents one standard deviation below the mean, so, the percentage of observations below 8 is 16%. Therefore the percentage of the calls lasted less than 8 min is 16%.
What is 5|3t+5|=25 and explain
Answer:
t=0 t = -10/3
Step-by-step explanation:
5|3t+5|=25
Divide each side by 5
5|3t+5| /5=25/5
|3t+5|=5
Now to get rid of the absolute value we get two equations, one positive and one negative
3t+5 =5 3t+5 = -5
Subtract 5 from each side
3t+5-5 =5-5 3t+5-5 = -5-5
3t =0 3t = -10
Divide by 3
3t/3 = 0/3 3t/3 = -10/3
t=0 t = -10/3
(used parentheses due to "forbidden language")
The figure Shows triangle ABC with medians (A F), BD, and CE. Segment (A F) is extended to H in such a way that segment GH is congruent to segment AG.
Which conclusion can be made based on the given conditions?
A) Segment GF is congruent to segment EG
B) Segment GF is half the length of segment EB
C) Segment GD is congruent to segment EG
D) Segment GD is half the length of segment HC
Answer:
Segment GD is half the length of segment HC ⇒ answer D
Step-by-step explanation:
* Look to the attached file
Answer: D) Segment GD is half the length of segment HC
What is the first step in sketching the graph of a rational function?
Answer:
Step-by-step explanation:
Find the x-value that makes the denominator zero. This x = a is the equation of the vertical asymptote. Next, determine the behavior of the function as x increases without bound in either direction. If there is a limiting value, then this y = d is the horizontal asymptote.
Consider the rational function
[tex]f(x)=\frac{P(x)}{Q(x)}[/tex]
We will find the Domain of Rational function first, means those value of rational function for which f(x) is defined, To do this we will evaluate those point first for which, Q(x)=0.
So, The first Step is "Finding Domain of the rational function" as well as the point where function is not defined.
⇒Consider the function
[tex]f(x)=\frac{x-3}{x-2}[/tex]
→Domain of the function is
x-2=0
x=2
=All Real Numbers , except at x=2.
=R- {2}
What is the length of BC in the right triangle below?
By the Pythagorean theorem [tex]a^2+b^2=c^2[/tex], the length of BC [tex]=\sqrt{22^2+120^2}=122[/tex]
Answer: D. 122
Step-by-step explanation: The given polygon is a right triangle, and to calculate any length of any side in a triangle we use Pythagorean Theorem. According to Pythagorean Theorem, the right triangle consists of one hypotenuse that is opposite to the right angle and two other sides that lie at an right angle. From this comes the relationship that always applies to the right triangles, that the area of the square located on the hypotenuse is equal to the sum of the areas of the squares that are located on the other two sides.
Pythagorean Theorem can also be expressed using the equation:
a² + b² = c²
where c is the hypotenuse and a and b are the other two sides of the right triangle.
According to the given triangle, the length of the BC side is the length of the hypotenuse we are looking for, while the lengths of the sides a and b are given, a = 22 and b = 120.
Using Pythagorean Theorem, hypotenuse BC is equal to
c² = a² + b² = 22² + 120² ⇒ c = √(22² + 120²) = √(484 + 14400) = √14884
c = 122
A 10-foot board is to be cut into 3 pieces. Two of the pieces will be the same length and one piece will be 2 feet longer than the other two.
Answer:
Step-by-step explanation:
According to the given statement two pieces are of same length:
If the length of one piece is x,
Then the length of other piece is also x.
And one piece is 2 feet longer than the other two = x+2
Total length of a board = 10 foot
Now make the equation from these terms:
x+x+x+2= 10
This is the equation of the given question.
You can further solve this equation:
x+x+x+2=10
3x+2=10
Now combine the constants:
3x=10-2
3x=8
x=8/3
x=2.67
It means that the length of two pieces of same length is 2.67
And the length of one piece which is longer than the other two = x+2 = 2.67+2 = 4.67 ....
Final answer:
To cut a 10-foot board into three pieces where two are the same length and one is 2 feet longer, denote the shorter length as 'x', create the equation 2x + (x + 2) = 10, and solve for 'x'. The two shorter pieces will each be approximately 2.67 feet and the longer one will be approximately 4.67 feet.
Explanation:
The question involves dividing a 10-foot board into three pieces with one piece being 2 feet longer than the other two equal pieces. To solve this, let's denote the length of the shorter pieces as 'x'. Since there are two of these, we have '2x', and the longer piece would be 'x + 2' feet long. The sum of the lengths of all three pieces is equal to the length of the board, so:
2x + (x + 2) = 10
This simplifies to:
3x + 2 = 10
Subtracting 2 from both sides gives:
3x = 8
Dividing both sides by 3 gives:
x = 8/3 or approximately 2.67 feet.
Therefore, the two shorter pieces are each approximately 2.67 feet long and the longer piece is 2.67 + 2, which is approximately 4.67 feet long.
Please help :) idk what the answer is
Answer:
[tex]\large\boxed{\dfrac{4}{3}}[/tex]
Step-by-step explanation:
Look at the picture.
The formula of a slope:
[tex]m=\dfrac{y_2-y_1}{x_2-x_1}[/tex]
From the graph we have two points (0, -4) and (3, 0).
Substitute:
[tex]m=\dfrac{0-(-4)}{3-0}=\dfrac{4}{3}[/tex]
:
Allen is building birdhouses that require 12-ft-long boards. How many pieces that are exactly 12ft long can be made from a board that is 814ft long?
Answer:
67 boards
Step-by-step explanation:
Given :
Total board length = 814 ft
Each board must be exactly 12 feet
Number of 12 ft boards which can be cut from 814 feet,
= 814 ÷ 12
= 67.83 boards
But because the question requires boards which are exactly 12 feet long, so we have to round down to the nearest whole number
hence 67.83 boards rounded down to the nearest whole board becomes 67 boards.
help asap pls
there is a 90% chance that a person eats dinner, a 60% chance a person eats dessert, and 50% chance the person will eat dinner and dessert. which of the following is true
Answer:
Eating dinner and eating dessert are dependent events because
P(dinner) . P(dessert) = 0.9 × 0.6 = 0.54 which is not equal to
P(dinner and desert) = 0.5 ⇒ answer A
Step-by-step explanation:
* Lets study the meaning independent and dependent probability
- Two events are independent if the result of the second event is not
affected by the result of the first event
- If A and B are independent events, the probability of both events
is the product of the probabilities of the both events
- P (A and B) = P(A) · P(B)
* Lets solve the question
∵ There is a 90% chance that a person eats dinner
∴ P(eating dinner) = 90/100 = 0.9
∵ There is a 60% chance a person eats dessert
∴ P(eating dessert) = 60/100 = 0.6
- If eating dinner and dating dessert are independent events, then
probability of both events is the product of the probabilities of the
both events
∵ P(eating dinner and dessert) = P(eating dinner) . P(eating dessert)
∴ P(eating dinner and dessert) = 0.9 × 0.6 = 0.54
∵ There is a 50% chance the person will eat dinner and dessert
∴ P(eating dinner and dessert) = 50/100 = 0.5
∵ P(eating dinner and dessert) ≠ P(eating dinner) . P(eating dessert)
∴ Eating dinner and eating dessert are dependent events because
P(dinner) . P(dessert) = 0.9 × 0.6 = 0.54 which is not equal to
P(dinner and desert) = 0.5
What is the order of rotational symmetry for the figure
Answer:
4
Step-by-step explanation:
First of all we will define rotational symmetry.
Rotational symmetry is when a shape looks the same after some rotation or less than one rotation.
The order of rotational symmetry is how many times it matches the original shape during the rotation.
So for the given shape, we can observe that the shape has four same type of sides. The shape in rotation will be like original shape 4 times in a complete rotation. So the order of symmetry is 4 ..
short cut method to mulitply 12 by 50
Answer:
12 * 50 = 600
Step-by-step explanation:
We have to multiply 12 by 50
Short cut method for multiplying a number by 50
Step 1: Take the half of the given number
Step 2: Multiply the result by 100
To find the short cut method to multiply 12 by 50
Here the number is 12
Step 1: divide the number by 2
12/2 = 6
Step 2: Multiply 6 by 100 we get 600
Therefore 12 * 50 = 600
Answer:
12 x 50= 600
Step-by-step explanation:
One way you can do is long and short. But the best way is to do it long because than you will get a better score and get the answer still.
[WILL GIVE BRAINLIEST ANSWER TO ANYONE WHO SOLVES FIRST]
Answer:
it is 4x=-9
Step-by-step explanation:
Use the quadratic expression 15x2+14x−16 to answer the questions.
A: Which statement describes the correct method to factor the quadratic expression?
B: What are the factors of the quadratic expression?
Select one answer for question A, and select two answers for question B.
A: This quadratic expression can be factored by using the difference of squares pattern.
A: This quadratic expression can be factored by finding the correct pair of binomial factors.
A: This quadratic expression can be factored by using the perfect square trinomial pattern.
B: (3x+4)
B: (3x−2)
B: (5x+8)
B: (5x+4)
B: (5x−4)
B: (3x+4)
Answer:
A: second one or middle choice
B: (5x + 8)(3x - 2)
Step-by-step explanation:
First question (A)
It is a trinomial. You can't use the difference of squares on it. The difference of squares have two terms as an answer.
It is not a perfect square. 15 is not a perfect square and c would have to be positive not minus to even think about using a perfect square.
So the A answer is the second one. You need two different binomials to factor this.
Second Question
You could try all the possible pairings and solve them by brute force.
The are five choices the go with the first one (3x + 4). Eventually you would get the answer, you would have to try 5 + 4 + 3 + 2 + 1 = 15 attempts.
There must be a shorter way.
14 means that the expressions are fairly far apart. This was just luck on my part. There is no logic. 8 and 2 for 16 are fairly far apart.
(5x + 8)(3x - 2)
The middle term is 8*3x - 2*5x = 14x and that looks like the answer.
PLEASE HELP ME ASAP 25 POINTS !!!!
Answer:
78˚
Step-by-step explanation:
Triangle MNP is congruent to Triangle QST, and so their angle measures are the same.
If you look at the order of the letters in the triangle names, you will notice that angle N lines up with angle S, so that means their angle measures are the same. Therefore if angle N is 78˚, angle S will be 78˚ as well.
Answer:
∠S = 78°
Step-by-step explanation:
Corresponding angles are equal, that is
∠Q = ∠M = 66° and
∠S = ∠N = 78°
Write the sum using summation notation, assuming the suggested pattern continues.
25 + 36 + 49 + 64 + ... + n2 + ...
Answer:
[tex]\sum_{n=5}^{\infty}n^2[/tex]
Step-by-step explanation:
The pattern given is:
25+36+49+64+...+n^2+...
The pattern can be written as
(5)^2+(6)^2+(7)^2+(8)^2+.....+n^2+....
The series is started with 5 and it continues up to infinity.
The summation notation for the given series is:
[tex]\sum_{n=5}^{\infty} n^2[/tex]
n= 1 and goes up to infinity and the series is made up of taking square of n,
The sum using summation notation, assuming the suggested pattern continues is :
[tex]25 + 36 + 49 + 64 + ... + n^2 + ...=\sum_{n=5}^{\infty} n^2[/tex]
Step-by-step explanation:We are given a series of numbers as
25 + 36 + 49 + 64 + ... + n^2 + ...
To write the sum using summation notation means we need to express this series in terms of a general n such that there is a whole summation expressing this series.
Here we see that each of the numbers could be expressed as follows:
[tex]25=5^2\\\\36=6^2\\\\49=7^2\\\\64=8^2[/tex]
and so on.
i.e. the series starts by taking the square of 5 then of 6 then 7 and so on.
and the series goes to infinity.
Hence, the summation notation will be given by:
[tex]25 + 36 + 49 + 64 + ... + n^2 + ...=\sum_{n=5}^{\infty} n^2[/tex]
if 3k is an even number integer which of the following cannot be an integer ?
A : k
B : k - 1
C : k/2
D : 3k
Answer:
c and a
because 3k has to add up the an even number so k by it self can not be an even number, and k/2 would still be an odd number because an off number divided by a even number still makes it odd
If 3k is an even integer then k is an even integer, and k and k - 1 are also integers. However, k/2 cannot be assured to be an integer unless k is a multiple of 4, making it the correct answer for what cannot be an integer.
If 3k is an even number, then k must also be an even number because an even number divided by 3 still leaves k as an integer. So, we can infer that k is an integer.
The option k - 1 must also be an integer because if k is an integer, subtracting 1 from it will result in another integer. The option 3k is given as an even integer, to begin with. However, the option k/2 cannot be an integer unless k is not only even but also a multiple of 4, because when you divide an even number that is not a multiple of 4 by 2, the result is not an integer.
Given the information provided in the question, we can conclude that k/2 is the option that cannot definitely be an integer without additional information specifying that k is indeed a multiple of 4.
Which equation can be used to find the volume of a sphere that has a radius of 9 inches
The volume of a sphere that has a radius of 9 inches is 3053.63in³.
V≈3053.63in³
V=4
3πr3=4
3·π·93≈3053.62806in³
Answer:
Therefore, C ) [tex]\frac{4*pi}{3} (9)^{3}[/tex].
Step-by-step explanation:
Given : A sphere that has a radius of 9 inches.
To find : Which equation can be used to find the volume of a sphere.
Solution: We have given
radius of sphere = 9 inches.
Volume of sphere = [tex]\frac{4*pi}{3} (radius)^{3}[/tex].
Plug the valu radius = 9 inches .
Volume of sphere = [tex]\frac{4*pi}{3} (9)^{3}[/tex].
Then equation c is correct answere.
Therefore, C ) [tex]\frac{4*pi}{3} (9)^{3}[/tex].
What is the solution to the equation below? Round your answer to two
decimal places.
2^3x=91
Answer:
The solution of the equation is [tex]x=2.169..[/tex]
Step-by-step explanation:
Given : Equation [tex]2^{3x}=91[/tex]
To find : What is the solution to the equation below?
Solution :
Equation [tex]2^{3x}=91[/tex]
Taking log both side,
[tex]\log(2^{3x})=\log(91)[/tex]
Apply logarithmic property, [tex]\log a^x=x\log a[/tex]
[tex]3x\log(2)=\log(91)[/tex]
[tex]3x=\frac{\log(91)}{\log(2)}[/tex]
[tex]x=\frac{\log(91)}{3\log(2)}[/tex]
[tex]x=2.169..[/tex]
Therefore, The solution of the equation is [tex]x=2.169..[/tex]
Which constants can be multiplied by the equations so one variable will be eliminated when the systems are added together?
5x + 13y = 232
12x + 7y = 218
A.The first equation can be multiplied by –13 and the second equation by 7 to eliminate y.
B.The first equation can be multiplied by 7 and the second equation by 13 to eliminate y.
C.The first equation can be multiplied by –12 and the second equation by 5 to eliminate x.
D.The first equation can be multiplied by 5 and the second equation by 12 to eliminate x.
Answer:
C
Step-by-step explanation:
So the system is:
5x+13y=232
12x+7y=218
------------------
Let's look at the options to see which will work:
A) Multiply first equation by -13: -65x-169y=-3016
& second equation by 7 : 84x+49y=1526
There are no opposites in the either one of the variable columns so not this option.
B) Multiply first equation by 7: 35x+91y=1624
& second equation by 13 : 156x+91y=2834
There are no opposites in the either one of the variable columns so this is not option unless we were asked to subtract the equations.
C) Multiply first equation by -12: -60x-156y=-2784
& second equation by 5 : 60x+35y=1090
There is a column that contains opposites here so when you add the equations the x-variable will get eliminated.
Answer:
3rd one
Step-by-step explanation: